How to Write the AP Lang Argument Essay (With Example)

December 14, 2023

ap lang argument essay example

We’d like to let you in on a little secret: no one, including us, enjoys writing timed essays. But a little practice goes a long way. If you want to head into your AP English Exam with a cool head, you’ll want to know what you’re getting into ahead of time. We can’t promise the AP Lang Argument Essay will ever feel like an island vacation, but we do have tons of hand tips and tricks (plus a sample essay!) below to help you do your best. This article will cover: 1) What is the AP Lang Argumentative Essay? 2) AP Lang Argument Rubric 3) AP Lang Argument Sample Prompt 4) AP Lang Argument Essay Example 5) AP Lang Argument Essay Example: Answer Breakdown.

What is the AP Lang Argument Essay?

The AP Lang Argument Essay is one of three essays included in the written portion of the AP English Exam. The full AP English Exam is 3 hours and 15 minutes long, with the first 60 minutes dedicated to multiple-choice questions. Once you complete the multiple-choice section, you move on to three equally weighted essays that ask you to synthesize, analyze, and interpret texts and develop well-reasoned arguments. The three essays include:

Synthesis essay: You’ll review various pieces of evidence and then write an essay that synthesizes (aka combines and interprets) the evidence and presents a clear argument. Read our write-up on How to Write the AP Lang Synthesis Essay here.

Argumentative essay: You’ll take a stance on a specific topic and argue your case.

Rhetorical essay: You’ll read a provided passage, then analyze the author’s rhetorical choices and develop an argument that explains why the author made those rhetorical choices. Read our write-up on How to Write the AP Lang Rhetorical Essay here.

AP Lang Argument Essay Rubric

The AP Lang Argument Essay is graded on 3 rubric categories : Thesis, Evidence and Commentary, and Sophistication . How can you make sure you cover all three bases in your essay? We’ll break down each rubric category with dos and don’ts below:

  • Thesis (0-1 point)

When it comes to grading your thesis, AP Exam graders are checking off a box: you either have a clear thesis or you don’t. So, what crucial components of a thesis will get you your check mark?

  • Make sure your thesis argues something . To satisfy your graders, your thesis needs to take a clear stance on the issue at hand.
  • Include your thesis statement in your intro paragraph. The AP Lang Argumentative essay is just that: an essay that makes an argument, so make sure you present your argument right away at the end of your first paragraph.
  • A good test to see if you have a thesis that makes an argument for your AP Lang Argumentative Essay: In your head, add the phrase “I agree/disagree that…” to the beginning of your thesis. If what follows doesn’t logically flow after that phrase (aka if what follows isn’t an agreement or disagreement), it’s likely you’re not making an argument.
  • In your thesis, outline the evidence you’ll cover in your body paragraphs.

AP Lang Argument Essay Rubric (Continued)

  • Avoid a thesis that merely restates the prompt.
  • Avoid a thesis that summarizes the text but does not make an argument.
  • Avoid a thesis that weighs the pros and cons of an issue. Your job in your thesis is to pick a side and stick with it.
  • Evidence and Commentary (0-4 points)

This rubric category is graded on a scale of 0-4 where 4 is the highest grade. Unlike the rhetorical and synthesis essays, the evidence you need to write your AP Lang Argument Essay is not provided to you. Rather, you’ll need to generate your own evidence and comment upon it.

What counts as evidence?

Typically, the AP Lang Argument Essay prompt asks you to reflect on a broad cultural, moral, or social issue that is open to debate. For evidence, you won’t be asked to memorize and cite statistics or facts. Rather, you’ll want to bring in real-world examples of:

  • Historical events
  • Current-day events from the news
  • Personal anecdotes

For this essay, your graders know that you’re not able to do research to find the perfect evidence. What’s most important is that you find evidence that logically supports your argument.

What is commentary?

In this essay, it’s important to do more than just provide examples relevant evidence. After each piece of evidence you include, you’ll need to explain why it’s significant and how it connects to your main argument. The analysis you include after your evidence is commentary .

  • Take a minute to brainstorm evidence that logically supports your argument. If you have to go out of your way to find the connection, it’s better to think of different evidence.
  • Include multiple pieces of evidence. There is no magic number, but do make sure you incorporate more than a couple pieces of evidence that support your argument.
  • Make sure you include more than one example of evidence, too. Let’s say you’re working on an essay that argues that people are always stronger together than apart. You’ve already included an example from history: during the civil rights era, protestors staged group sit-ins as a powerful form of peaceful protest. That’s just one example, and it’s hard to make a credible argument with just one piece of evidence. To fix that issue, think of additional examples from history, current events, or personal experience that are not related to the civil rights era.
  • After you include each piece of evidence, explain why it’s significant and how it connects to your main argument.
  • Don’t summarize or speak generally about the topic. Everything you write must be backed up with specific and relevant evidence and examples.
  • Don’t let quotes speak for themselves. After every piece of evidence you include, make sure to explain and connect the evidence to your overarching argument.

AP Lang Argument Essay (Continued)

  • Sophistication (0-1 point)

According to the College Board , one point can be awarded to AP Lang Argument essays that achieve a high level of sophistication. You can accomplish that in four ways:

  • Crafting a nuanced argument by consistently identifying and exploring complexities or tensions.
  • Articulating the implications or limitations of an argument by situating it within a broader context.
  • Making effective rhetorical choices that consistently strengthen the force and impact of the student’s argument.
  • Employing a style that is consistently vivid and persuasive.

In sum, this means you can earn an additional point for going above and beyond in depth, complexity of thought, or by writing an especially persuasive, clear, and well-structured essay. In order to earn this point, you’ll first need to do a good job with the fundamentals: your thesis, evidence, and commentary. Then, to earn your sophistication point, follow these tips:

  • Outline your essay before you begin to ensure it flows in a clear and cohesive way.
  • Include well-rounded evidence. Don’t rely entirely on personal anecdotes, for example. Incorporate examples from current events or history, as well.
  • Thoroughly explain how each piece of evidence connects to your thesis in order to fully develop your argument.
  • Explore broader implications. If what you’re arguing is true, what does that mean to us today? Who is impacted by this issue? What real-world issues are relevant to this core issue?
  • Briefly explore the other side of the issue. Are the instances where your argument might not be true? Acknowledge the other side, then return to proving your original argument.
  • Steer clear of generalizations (avoid words like “always” and “everyone”).
  • Don’t choose an argument you can’t back up with relevant examples.
  • Avoid complex sentences and fancy vocabulary words unless you use them often. Long, clunky sentences with imprecisely used words are hard to follow.

AP Lang Argument Sample Prompt

The sample prompt below is published online by the College Board and is a real example from the 2021 AP English Exam. The prompt provides background context, essay instructions, and the text you need to analyze.

Suggested time—40 minutes.

Many people spend long hours trying to achieve perfection in their personal or professional lives. Similarly, people often demand perfection from others, creating expectations that may be challenging to live up to. In contrast, some people think perfection is not attainable or desirable.

Write an essay that argues your position on the value of striving for perfection.

In your response you should do the following:

  • Respond to the prompt with a thesis that presents a defensible position.
  • Provide evidence to support your line of reasoning.
  • Explain how the evidence supports your line of reasoning.
  • Use appropriate grammar and punctuation in communicating your argument.

AP Lang Argument Essay Example

As the old phrase says, “Practice makes perfect.” But is perfection something that is actually attainable? Sometimes, pushing for perfection helps us achieve great things, but most often, perfectionism puts too much pressure on us and prevents us from knowing when we have done the best we can. Striving for perfection can only lead us to shortchange ourselves. Instead, we should value learning, growth, and creativity and not worry whether we are first or fifth best.

Students often feel the need to be perfect in their classes, and this can cause students to struggle or stop making an effort in class. In elementary and middle school, for example, I was very nervous about public speaking. When I had to give a speech, my voice would shake, and I would turn very red. My teachers always told me “relax!” and I got Bs on Cs on my speeches. As a result, I put more pressure on myself to do well, spending extra time making my speeches perfect and rehearsing late at night at home. But this pressure only made me more nervous, and I started getting stomach aches before speaking in public.

Once I got to high school, however, I started doing YouTube make-up tutorials with a friend. We made videos just for fun, and laughed when we made mistakes or said something silly. Only then, when I wasn’t striving to be perfect, did I get more comfortable with public speaking.

AP Lang Argumentative Essay Example (Continued)

In the world of art and business and science, perfectionism can also limit what we are able to achieve. Artists, for example, have to take risks and leave room for creativity. If artists strive for perfection, then they won’t be willing to fail at new experiments and their work will be less innovative and interesting. In business and science, many products, like penicillin for example, were discovered by accident. If the scientist who discovered penicillin mold growing on his petri dishes had gotten angry at his mistake and thrown the dishes away, he would never have discovered a medicine that is vital to us today.

Some fields do need to value perfection. We wouldn’t like it, for example, if our surgeon wasn’t striving for perfection during our operation. However, for most of us, perfectionism can limit our potential for learning and growth. Instead of trying to be perfect, we should strive to learn, innovate, and do our personal best.

AP Lang Argument Essay Example: Answer Breakdown

The sample AP Lang Argumentative Essay above has some strengths and some weaknesses. Overall, we would give this essay a 3 or a 4. Let’s break down what’s working and what could be improved:

  • The essay offers a thesis that makes a clear argument that is relevant to the prompt: “Striving for perfection can only lead us to shortchange ourselves. Instead, we should value learning, growth, and creativity and not worry whether we are first or fifth best.”
  • The first body paragraph provides evidence that supports the essay’s thesis. This student’s personal anecdote offers an example of a time when perfectionism led them to shortchange themselves.
  • The second body paragraph provides additional evidence that supports the essay’s thesis. The example describing the discovery of penicillin offers another example of a situation in which perfectionism might have limited scientific progress.
  • The writer offers commentary explaining how her examples of public speaking and penicillin illustrate that we should “value learning, growth, and creativity” over perfectionism.
  • The essay follows one line of reasoning and does not stray into tangents.
  • The essay is organized well with intro, body, and concluding paragraphs. Overall, it is easy to read and is free of grammar errors.

What could be improved:

  • Although the second body paragraph provides one good specific example about the discovery of penicillin, the other examples it offers about art and business are only discussed generally and aren’t backed up with evidence. This paragraph would be stronger if it provided more examples. Or, if this writer couldn’t think of examples, they could have left out mentions of art and business altogether and included alternate evidence instead.
  • This writer would more thoroughly support their argument if they were able to offer one more example of evidence. They could provide another personal anecdote, an example from history, or an example from current events.
  • The writer briefly mentions the other side of the argument in their concluding paragraph: “Some fields do need to value perfection. We wouldn’t like it, for example, if our surgeon wasn’t striving for perfection during our operation.” Since it’s so brief a mention of the other side, it undermines the writer’s overall argument. This writer should either dedicate more time to reflecting on why even surgeons should “value learning, growth, and creativity” over perfectionism, or they should leave these sentences out.

AP Lang Argument Essay Example—More Resources

Looking for more tips to help you master your AP Lang Argumentative Essay? Brush up on 20 Rhetorical Devices High School Students Should Know and read our Tips for Improving Reading Comprehension .

If you’re ready to start studying for another part of the AP English Exam, find more expert tips in our How to Write the AP Lang Synthesis and How to Write the AP Lang Rhetorical Essay blog posts.

  • High School Success

' src=

Christina Wood

Christina Wood holds a BA in Literature & Writing from UC San Diego, an MFA in Creative Writing from Washington University in St. Louis, and is currently a Doctoral Candidate in English at the University of Georgia, where she teaches creative writing and first-year composition courses. Christina has published fiction and nonfiction in numerous publications, including The Paris Review , McSweeney’s , Granta , Virginia Quarterly Review , The Sewanee Review , Mississippi Review , and Puerto del Sol , among others. Her story “The Astronaut” won the 2018 Shirley Jackson Award for short fiction and received a “Distinguished Stories” mention in the 2019 Best American Short Stories anthology.

  • 2-Year Colleges
  • Application Strategies
  • Best Colleges by Major
  • Best Colleges by State
  • Big Picture
  • Career & Personality Assessment
  • College Essay
  • College Search/Knowledge
  • College Success
  • Costs & Financial Aid
  • Dental School Admissions
  • Extracurricular Activities
  • Graduate School Admissions
  • High Schools
  • Law School Admissions
  • Medical School Admissions
  • Navigating the Admissions Process
  • Online Learning
  • Private High School Spotlight
  • Summer Program Spotlight
  • Summer Programs
  • Test Prep Provider Spotlight

College Transitions Sidebar Block Image

“Innovative and invaluable…use this book as your college lifeline.”

— Lynn O'Shaughnessy

Nationally Recognized College Expert

College Planning in Your Inbox

Join our information-packed monthly newsletter.

I am a... Student Student Parent Counselor Educator Other First Name Last Name Email Address Zip Code Area of Interest Business Computer Science Engineering Fine/Performing Arts Humanities Mathematics STEM Pre-Med Psychology Social Studies/Sciences Submit

What are your chances of acceptance?

Calculate for all schools, your chance of acceptance.

Duke University

Your chancing factors

Extracurriculars.

argumentative essay prompts for ap lang

How to Write the AP Lang Argument Essay + Examples

What’s covered:, what is the ap language argument essay, tips for writing the ap language argument essay, ap english language argument essay examples, how will ap scores impact my college chances.

In 2023, over 550,148 students across the U.S. took the AP English Language and Composition Exam, and 65.2% scored higher than a 3. The AP English Language Exam tests your ability to analyze a piece of writing, synthesize information, write a rhetorical essay, and create a cohesive argument. In this post, we’ll be discussing the best way to approach the argumentative essay section of the test, and we’ll give you tips and tricks so you can write a great essay.

The AP English Language Exam as of 2023 is structured as follows:

Section 1: 45 multiple choice questions to be completed in an hour. This portion counts for 45% of your score. This section requires students to analyze a piece of literature. The questions ask about its content and/or what could be edited within the passage.

Section 2: Three free response questions to be completed in the remaining two hours and 15 minutes. This section counts for 55% of your score. These essay questions include the synthesis essay, the rhetorical essay, and the argumentative essay.

  • Synthesis essay: Read 6-7 sources and create an argument using at least three of the sources.
  • Rhetorical analysis essay: Describe how a piece of writing evokes meaning and symbolism.
  • Argumentative essay: Pick a side of a debate and create an argument based on evidence. In this essay, you should develop a logical argument in support of or against the given statement and provide ample evidence that supports your conclusion. Typically, a five paragraph format is great for this type of writing. This essay is scored holistically from 1 to 9 points.

Do you want more information on the structure of the full exam? Take a look at our in-depth overview of the AP Language and Composition Exam .

Although the AP Language Argument may seem daunting at first, once you understand how the essay should be structured, it will be a lot easier to create cohesive arguments.

Below are some tips to help you as you write the essay.

1. Organize your essay before writing

Instead of jumping right into your essay, plan out what you will say beforehand. It’s easiest to make a list of your arguments and write out what facts or evidence you will use to support each argument. In your outline, you can determine the best order for your arguments, especially if they build on each other or are chronological. Having a well-organized essay is crucial for success.

2. Pick one side of the argument, but acknowledge the other side

When you write the essay, it’s best if you pick one side of the debate and stick with it for the entire essay. All your evidence should be in support of that one side. However, in your introductory paragraph, as you introduce the debate, be sure to mention any merit the arguments of the other side has. This can make the essay a bit more nuanced and show that you did consider both sides before determining which one was better. Often, acknowledging another viewpoint then refuting it can make your essay stronger.

3. Provide evidence to support your claims

The AP readers will be looking for examples and evidence to support your argument. This doesn’t mean that you need to memorize a bunch of random facts before the exam. This just means that you should be able to provide concrete examples in support of your argument.

For example, if the essay topic is about whether the role of the media in society has been detrimental or not, and you argue that it has been, you may talk about the phenomenon of “fake news” during the 2016 presidential election.

AP readers are not looking for perfect examples, but they are looking to see if you can provide enough evidence to back your claim and make it easily understood.

4. Create a strong thesis statement

The thesis statement will set up your entire essay, so it’s important that it is focused and specific, and that it allows for the reader to understand your body paragraphs. Make sure your thesis statement is the very last sentence of your introductory paragraph. In this sentence, list out the key points you will be making in the essay in the same order that you will be writing them. Each new point you mention in your thesis should start a paragraph in your essay.

Below is a prompt and sample student essay from the May 2019 exam . We’ll look at what the student did well in their writing and where they could improve.

Prompt: “The term “overrated” is often used to diminish concepts, places, roles, etc. that the speaker believes do not deserve the prestige they commonly enjoy; for example, many writers have argued that success is overrated, a character in a novel by Anthony Burgess famously describes Rome as a “vastly overrated city,” and Queen Rania of Jordan herself has asserted that “[b]eing queen is overrated.”

Select a concept, place, role, etc. to which you believe that the term “overrated” should be applied. Then, write a well-developed essay in which you explain your judgment. Use appropriate evidence from your reading, experience, or observations to support your argument.

Sample Student Essay #1:

[1] Competition is “overrated.” The notion of motivation between peers has evolved into a source of unnecessary stress and even lack of morals. Whether it be in an academic environment or in the industry, this new idea of competition is harmful to those competing and those around them.

[2] Back in elementary school, competition was rather friendly. It could have been who could do the most pushups or who could get the most imaginary points in a classroom for a prize. If you couldn’t do the most pushups or win that smelly sticker, you would go home and improve yourself – there would be no strong feelings towards anyone, you would just focus on making yourself a better version of yourself. Then as high school rolled around, suddenly applying for college doesn’t seem so far away –GPA seems to be that one stat that defines you – extracurriculars seem to shape you – test scores seem to categorize you. Sleepless nights, studying for the next day’s exam, seem to become more and more frequent. Floating duck syndrome seems to surround you (FDS is where a competitive student pretends to not work hard but is furiously studying beneath the surface just like how a duck furiously kicks to stay afloat). All of your competitors appear to hope you fail – but in the end what do you and your competitor’s gain? Getting one extra point on the test? Does that self-satisfaction compensate for the tremendous amounts of acquired stress? This new type of “competition” is overrated – it serves nothing except a never-ending source of anxiety and seeks to weaken friendships and solidarity as a whole in the school setting.

[3] A similar idea of “competition” can be applied to business. On the most fundamental level, competition serves to be a beneficial regulator of prices and business models for both the business themselves and consumers. However, as businesses grew increasingly greedy and desperate, companies resorted to immoral tactics that only hurt their reputations and consumers as a whole. Whether it be McDonald’s coupons that force you to buy more food or tech companies like Apple intentionally slowing down your iPhone after 3 years to force you to upgrade to the newest device, consumers suffer and in turn speak down upon these companies. Similar to the evolved form of competition in school, this overrated form causes pain for all parties and has since diverged from the encouraging nature that the principle of competition was “founded” on.

The AP score for this essay was a 4/6, meaning that it captured the main purpose of the essay but there were still substantial parts missing. In this essay, the writer did a good job organizing the sections and making sure that their writing was in order according to the thesis statement. The essay first discusses how competition is harmful in elementary school and then discusses this topic in the context of business. This follows the chronological order of somebody’s life and flows nicely.

The arguments in this essay are problematic, as they do not provide enough examples of how exactly competition is overrated. The essay discusses the context in which competition is overrated but does not go far enough in explaining how this connects to the prompt.

In the first example, school stress is used to explain how competition manifests. This is a good starting point, but it does not talk about why competition is overrated; it simply mentions that competition can be unhealthy. The last sentence of that paragraph is the main point of the argument and should be expanded to discuss how the anxiety of school is overrated later on in life. 

In the second example, the writer discusses how competition can lead to harmful business practices, but again, this doesn’t reflect the reason this would be overrated. Is competition really overrated because Apple and McDonald’s force you to buy new products? This example could’ve been taken one step farther. Instead of explaining why business structures—such as monopolies—harm competition, the author should discuss how those particular structures are overrated.

Additionally, the examples the writer used lack detail. A stronger essay would’ve provided more in-depth examples. This essay seemed to mention examples only in passing without using them to defend the argument.

It should also be noted that the structure of the essay is incomplete. The introduction only has a thesis statement and no additional context. Also, there is no conclusion paragraph that sums up the essay. These missing components result in a 4/6.

Now let’s go through the prompt for a sample essay from the May 2022 exam . The prompt is as follows:

Colin Powell, a four-star general and former United States Secretary of State, wrote in his 1995 autobiography: “[W]e do not have the luxury of collecting information indefinitely. At some point, before we can have every possible fact in hand, we have to decide. The key is not to make quick decisions, but to make timely decisions.”

Write an essay that argues your position on the extent to which Powell’s claim about making decisions is valid. 

In your response you should do the following:

  • Respond to the prompt with a thesis that presents a defensible position. 
  • Provide evidence to support your line of reasoning. 
  • Explain how the evidence supports your line of reasoning. 
  • Use appropriate grammar and punctuation in communicating your argument.

Sample Student Essay #2:

Colin Powell, who was a four star general and a former United States Secretary of State. He wrote an autobiography and had made a claim about making decisions. In my personal opinion, Powell’s claim is true to full extent and shows an extremely valuable piece of advice that we do not consider when we make decisions.

Powell stated, “before we can have every possible fact in hand we have to decide…. but to make it a timely decision” (1995). With this statement Powell is telling the audience of his autobiography that it does not necessarily matter how many facts you have, and how many things you know. Being able to have access to everything possible takes a great amount of time and we don’t always have all of the time in the world. A decision has to be made with what you know, waiting for something else to come in while trying to make a decision whether that other fact is good or bad you already have a good amount of things that you know. Everyone’s time is valuable, including yours. At the end of the day the decision will have to be made and that is why it should be made in a “timely” manner.

This response was graded for a score of 2/6. Let’s break down the score to smaller points that signify where the student fell short.

The thesis in this essay is clearly outlined at the end of the first paragraph. The student states their agreement with Powell’s claim and frames the rest of their essay around this stance. The success in scoring here lies in the clear communication of the thesis and the direction the argument will take. It’s important to make the thesis statement concise, specific, and arguable, which the student has successfully done.

While the student did attempt to provide evidence to support their thesis, it’s clear that their explanation lacks specific detail and substance. They referenced Powell’s statement, but did not delve into how this statement has proven true in specific instances, and did not provide examples that could bring the argument to life.

Commentary is an essential part of this section’s score. It means explaining the significance of the evidence and connecting it back to the thesis. Unfortunately, the student’s commentary here is too vague and does not effectively elaborate on how the evidence supports their argument.

To improve, the student could use more concrete examples to demonstrate their point and discuss how each piece of evidence supports their thesis. For instance, they could discuss specific moments in Powell’s career where making a timely decision was more valuable than waiting for all possible facts. This would help illustrate the argument in a more engaging, understandable way.

A high score in the “sophistication” category of the grading rubric is given for demonstrating a complex understanding of the rhetorical situation (purpose, audience, context, etc.), making effective rhetorical choices, or establishing a line of reasoning. Here, the student’s response lacks complexity and sophistication. They’ve simply agreed with Powell’s claim and made a few general statements without providing a deeper analysis or effectively considering the rhetorical situation.

To increase sophistication, the student could explore possible counterarguments or complexities within Powell’s claim. They could discuss potential drawbacks of making decisions without all possible facts, or examine situations where timely decisions might not yield the best results. By acknowledging and refuting these potential counterarguments, they could add more depth to their analysis and showcase their understanding of the complexities involved in decision-making.

The student could also analyze why Powell, given his background and experiences, might have come to such a conclusion, thus providing more context and showing an understanding of the rhetorical situation.

Remember, sophistication in argumentation isn’t about using fancy words or complicated sentences. It’s about showing that you understand the complexity of the issue at hand and that you’re able to make thoughtful, nuanced arguments. Sophistication shows that you can think critically about the topic and make connections that aren’t immediately obvious.

Now that you’ve looked at an example essay and some tips for the argumentative essay, you know how to better prepare for the AP English Language and Composition Exam.

While your AP scores don’t usually impact your admissions chances , colleges do care a lot about your course rigor. So, taking as many APs as you can will certainly boost your chances! AP scores can be a way for high-performing students to set themselves apart, particularly when applying to prestigious universities. Through the process of self-reporting scores , you can show your hard work and intelligence to admissions counselors.

That said, the main benefit of scoring high on AP exams comes once you land at your dream school, as high scores can allow you to “test out” of entry-level requirements, often called GE requirements or distribution requirements. This will save you time and money.

To understand how your course rigor stacks up, check out CollegeVine’s free chancing engine . This resource takes your course rigor, test scores, extracurriculars, and more, to determine your chances of getting into over 1600 colleges across the country!

Related CollegeVine Blog Posts

argumentative essay prompts for ap lang

argumentative essay prompts for ap lang

Crafting an Impressive Argumentative Essay for AP Lang

argumentative essay prompts for ap lang

Writing an argumentative essay for AP Language and Composition requires a strategic approach to effectively convey your perspective. Here's a guide to crafting an impressive argumentative essay:

1. Understand the Prompt:

   - Tip: Carefully read and analyze the prompt. Identify the key elements, including the task you are asked to perform and any specific requirements or constraints.

2. Develop a Clear Thesis Statement:

   - Tip: Formulate a concise and focused thesis statement that clearly states your argument or position. This statement should guide the reader on what to expect in your essay.

3. Identify Your Audience:

   - Tip: Consider your target audience and tailor your language and arguments accordingly. Understanding your audience helps you establish a connection and effectively convey your message.

4. Organize Your Essay Effectively:

   - Tip: Structure your essay with a clear introduction, body paragraphs, and a conclusion. Ensure a logical flow of ideas, with each paragraph contributing to the overall argument.

5. Provide Context and Background:

   - Tip: Begin with a brief introduction that provides context for your argument. Clearly state the issue at hand and offer background information to help readers understand the significance of your argument.

6. Present a Strong Claim:

   - Tip: Clearly articulate your main claim or argument. This should be a debatable statement that forms the core of your essay. Avoid vague or overly broad claims.

7. Support Your Argument with Evidence:

   - Tip: Use relevant and compelling evidence to support your claims. This can include facts, statistics, examples, anecdotes, or quotations. Ensure that your evidence is credible and directly contributes to your argument.

8. Address Counterarguments:

   - Tip: Acknowledge potential counterarguments and address them in your essay. This demonstrates a nuanced understanding of the issue and strengthens your overall argument.

9. Use Persuasive Language:

    - Tip: Choose language that is persuasive and impactful. Utilize rhetorical devices, vivid imagery, and compelling language to engage your readers and convey the urgency or importance of your argument.

10. Craft Well-Reasoned Body Paragraphs:

    - Tip: Dedicate separate paragraphs to different aspects of your argument. Each paragraph should have a clear topic sentence, supporting evidence, and a concise explanation of how the evidence relates to your thesis.

11. Ensure Cohesive Transitions:

    - Tip: Use transitional phrases and sentences to ensure a smooth transition between paragraphs. This enhances the coherence of your essay and guides readers through your line of reasoning.

12. Conclude Effectively:

    - Tip: Summarize your main points in the conclusion and restate the significance of your argument. Avoid introducing new information in the conclusion. End with a strong closing statement that leaves a lasting impression.

13. Revise and Edit:

    - Tip: Set aside time for revision and editing. Review your essay for clarity, coherence, and grammatical accuracy. Ensure that your argument is compelling and well-supported.

14. Seek Feedback:

    - Tip: If possible, seek feedback from peers, teachers, or mentors. External perspectives can provide valuable insights and help you identify areas for improvement.

15. Stay Focused on the Argument:

    - Tip: Throughout the essay, stay focused on your central argument. Avoid unnecessary tangents or diverging from the main point. Every part of your essay should contribute to strengthening your overall argument.

Crafting an impressive argumentative essay for AP Language and Composition involves a combination of persuasive writing techniques, solid evidence, and effective organization. By following these tips, you can create a compelling essay that showcases your ability to engage in sophisticated argumentation and rhetorical analysis.

You Might Also Like

argumentative essay prompts for ap lang

Scholarship Application Process

Filling out college scholarship applications is indeed a time-consuming task that needs much effort and patience. This guide will help you through the entire scholarship application process

argumentative essay prompts for ap lang

How to Leverage your Summer to Boost Admissions

Summer programs offer students the chance to explore new areas, interests, and exciting fields. Here you can check some summer programs - Read our blog

argumentative essay prompts for ap lang

Know How to Build a Great College List

Want to choose best college for your study? Get some amazing guidelines that will help you to create a great college list for your admission - Read our blog

AP Guru has been helping students since 2010 gain admissions to their dream universities by helping them in their college admissions and SAT and ACT Prep

Free Resources

Tackling the AP English Language and Composition essays: part 1

Statistical Mediation & Moderation in Psychological Research-Apr-06-2021-08-55-30-55-PM

So, what are the three AP Lang Essays? The College Board shares a lot of general information about these essays on its website, as well as a large number of excellent sample essays. I suggest you take the time to review all of that material, here. But here’s my primer:

On the AP Lang Exam, there are three essays to write, all in a row (during the second half of the exam, after an initial multiple-choice portion). They are:

  • The Synthesis Essay: You’ll be given a general topic or question for debate (like: should public libraries continue to exist? Or: is eminent domain just?). Multiple short sources taking positions on that topic will follow the prompt. You will then be asked to write your own, short essay taking a position on the topic, citing at least three of the sources that you read.
  • The Rhetoric Essay: You’ll be given a short, rhetorically interesting passage, either taking a position on a topic, telling a story, or performing some other function. You will then be asked to write a short essay analyzing this passage’s use of language/rhetorical approach.
  • The Argument Essay: You will be given some position, usually stated in some brief excerpt from an author’s work. For example, you might be given an excerpt from Proust that suggests that people often regret their choices, or an excerpt from Eleanor Roosevelt praising the virtue of courage. You will then be asked to take your own position on the topic. This time, you won’t be given sources to help you make your arguments; all of your arguments must come from your own brain. 

The scoring rubric for each essay is roughly similar, with six possible points awarded: there is one point for argument, four points for evidence and analysis, and one point for “sophistication.” What this means is that, in brief, you need to do three things on every essay to get a perfect score: 

  • Have an argument.
  • Back up your argument with evidence and analyze how that evidence supports your argument.
  • Have an ineffable, excellent quality to your writing, a sort of dexterity of mind and language, for which the scorers have reserved one, sacred point. 

You can’t really control whether or not you can achieve #3, and a lot of that will be based on your prior level of experience writing/reading; but you can control whether or not you achieve #1-2. So, a high score is totally within your power! The TLDR version of this post is: make a clear argument and back it up with concrete, analyzed evidence. But, of course, that’s not as easy as it looks, and I have many more thoughts on how to actually achieve it, and achieve it well...

The six major components of successfully writing a timed essay on an exam are:

  • Organizing your time
  • Reading and Annotating
  • Outlining Part 1: Thesis
  • Outlining Part 2: Structure
  • Writing Part 1: Paragraphs (Intro, Evidence, Analysis, Conclusion)
  • Writing Part 2: Sentence by Sentence

#1 Organizing your time

On the AP Lang exam, you get a total of 2 hours and 15 minutes to write your three essays. This time is split into chunks. First, there is a 15 minute “reading period”; next, there is a 2 hour “writing period.” What this seems to imply is that the exam would like you to read all of the questions and their supplemental texts (the Synthesis Essay question and texts, the Rhetoric Essay question and passage, the Argument Essay question and short question blurb) in the 15 minute reading period, and then proceed to write the essays, in response, in the two hour writing period. This, however, is obviously an insane approach. For one thing, it’s kind of impossible: no one could keep the details of three different essay questions and associated readings together in their head all at once. For another, it’s really time inefficient: if you read all the material for all three essays first, you’re going to have to go back to it, a lot, each time you start to write a new essay, to jog your memory. Basically, no one in their right mind would (or does) advise this approach. And even the College Board seems to know it makes no sense, because they allow you to continue reading and referring to the questions and texts after the reading period. 

What you should do instead? Simply treat the whole 2 hours and 15 minutes as a single time block. Divide it into three units of 45 minutes. Then, read and answer each of the three questions one after the other, giving 45 minutes to each. Start with the Synthesis Essay, followed by the Rhetoric Essay, and then the Argument Essay. 

Your process should look like this: during the 15 minute reading period, begin work on the Synthesis Essay by reading the question and texts and planning that essay. Then, when the 2-hour timer starts, devote the first 30 minutes to actually writing that essay. Next, spend 45 minutes reading the Rhetoric Essay question and passage, and writing the Rhetoric Essay. Finally, spend the last 45 minutes reading the Argument Essay question/blurb and then writing the Argument Essay. The Argument Essay should actually take you less time than the first two, which means you should end up with 5-10 minutes to proofread your other essays. That said, I advise that you leave time at the end of each 45-minute block to check over each individual essay. 

Now let’s talk about the Rhetoric Essay in particular. How should you organize your 45 minutes here? I suggest mapping out your time roughly like this: take about ten minutes to read the passage, take notes, and brainstorm; then, take about five minutes to make an outline for your essay; next, take about twenty to twenty-five minutes to write. Leave an extra five to seven minutes at the end to re-read and edit your work. As you practice, you might notice that slightly different divisions of time work best for you – feel free to be flexible! You don’t have to stick to your timetable exactly . BUT you should try to stick to a version of this timetable so that you have enough time for each of the steps. How? Watch the clock!

#2 Reading and Annotating

The Rhetoric Essay asks you to analyze the language or rhetoric that a passage uses to achieve its ends. In your first ten minutes of reading, you should be keeping an eye out for two things: 

  • What is this passage trying to achieve? Is it trying to persuade the reader of an argument (often the case)? Is it trying to entertain the reader with a story (sometimes the case)? Is it trying to make the reader laugh? Is it trying to make the reader think? Identify the passage’s main purpose.
  • What rhetorical methods or devices does the passage use to achieve its aims? What exactly is it doing to achieve its aims? Yes, you should be watching out for rhetorical devices that already have fancy names, like “allusion” or “alliteration,” but you should also be using your OWN language/descriptive powers to identify the passage’s methods. You might, for example, note things like: “makes argument largely through anecdote” or “addresses counterarguments” or “lists so many absurd situations that they start to feel normal.” Try to identify not just rhetorical methods the passage uses, but also the central ones it uses.

To achieve this, I suggest proceeding as follows: read one paragraph. Once you’re done, stop, reflect, and note (in the margins) the most important rhetorical devices the passage used to achieve its aims (as far as you understand them thus far). Do this for each paragraph you read. Once you’re done, you should have a handy list in the margin of rhetorical tactics the passage uses. Which ones, looking back, seem to come up the most frequently? Which ones, even if they don’t come up frequently, seem particularly central to the passage’s aims? The tactics you identify will soon play a role in your essay’s thesis. 

Next, you’ll be ready to write an outline for your essay, mapping out (as best you can) its thesis and structure. In the next blog post , we’ll begin with that step. 

Related Content

pep

Find what you need to study

2024 AP English Language and Composition Exam Guide

12 min read • august 18, 2023

A Q

Your guide to the 2024 AP English Language and Composition exam

We know that studying for your AP exams can be stressful, but Fiveable has your back! We created a study plan to help you crush your AP English Language and Composition exam. This guide will continue to update with information about the 2024 exams, as well as helpful resources to help you do your best on test day.  Unlock Cram Mode  for access to our cram events—students who have successfully passed their AP exams will answer your questions and guide your last-minute studying LIVE! And don't miss out on unlimited access to our database of thousands of practice questions. FYI, something cool is coming your way Fall 2023! 👀

Format of the 2024 AP English Language and Composition exam

This year, all AP exams will cover all units and essay types. The 2024 AP English Language and Composition exam format will be:

Section I: Multiple Choice - 45% of your score

45 questions in 1 hour

Section II: Free Response Section - 55% of your score

2 hours and 15 minutes for:

1 synthesis essay

1 rhetorical analysis essay

1 argument essay

Scoring Rubric for the 2024 AP Lang Essays

Synthesis Essay

1 point for a defensible thesis that responds to the prompt

Evidence and Commentary

Max of 4 points for providing evidence from at least 3 sources that support the line of reasoning AND commentary that explains and analyzes the evidence

Sophistication

1 point any of the following:

Creating a nuanced argument

Showing the limitations of the argument

Making effective rhetorical choices

Employing a style that is vivid and persuasive

Rhetorical Analysis Essay

1 point for a defensible thesis that analyzes rhetorical choices

Max of 4 points for providing specific evidence AND consistently explaining how the evidence relates to the line of reasoning AND showing how the rhetorical choices contribute to the author's message .

1 point for any of the following:

Explaining the significance of the rhetorical choices ( rhetorical situation )

Explaining the complexities of the passage and their purpose

Argument Essay

1 point for a defensible thesis

Max of 4 points for providing specific evidence AND consistently explaining the relevance of that evidence .

Crafting a nuanced argument by identifying complexities

Explaining the limitations of the argument by placing it in a broader context

Making rhetorical choices to improve the argument

Check out our study plan below to find resources and tools to prepare for your AP English Language and Composition exam.

When is the 2024 AP English Language and Composition Exam and How Do I Take It?

How should i prepare for the ap lang exam.

First, take stock of your progress in the course so far. What areas have you excelled and which sections need more focus? Download the AP English Language Cheatsheet PDF - a single sheet that covers everything you need to know at a high level. Take note of your strengths and weaknesses!

Build your study plan to review every unit and question type, but focus most on the areas that need the most improvement and practice. We’ve put together this plan to help you study between now and May. This will cover all of the units and essay types to prepare you for your exam

Practice essays are your best friends! The more essays you write, the more automatic the process will come, and the easier the AP exam will be!

Try some of the past exam questions here

We've put together the study plan found below to help you study between now and May. This will cover all of the units and essay types to prepare you for your exam. Pay special attention to the units that you need the most improvement in.

Study, practice, and review for test day with other students during our live cram sessions via  Cram Mode . Cram live streams will teach, review, and practice important topics from AP courses, college admission tests, and college admission topics. These streams are hosted by experienced students who know what you need to succeed.

Pre-Work: Set Up Your Study Environment

Before you begin studying, take some time to get organized.

🖥 Create a study space.

Make sure you have a designated place at home to study. Somewhere you can keep all of your materials, where you can focus on learning, and where you are comfortable. Spend some time prepping the space with everything you need and you can even let others in the family know that this is your study space. 

📚 Organize your study materials.

Get your notebook, textbook, prep books, or whatever other physical materials you have. Also, create a space for you to keep track of review. Start a new section in your notebook to take notes or start a Google Doc to keep track of your notes. Get yourself set up!

📅 Plan designated times for studying.

The hardest part about studying from home is sticking to a routine. Decide on one hour every day that you can dedicate to studying. This can be any time of the day, whatever works best for you. Set a timer on your phone for that time and really try to stick to it. The routine will help you stay on track.

🏆 Decide on an accountability plan.

How will you hold yourself accountable to this study plan? You may or may not have a teacher or rules set up to help you stay on track, so you need to set some for yourself. First, set your goal. This could be studying for x number of hours or getting through a unit. Then, create a reward for yourself. If you reach your goal, then x. This will help stay focused!

🤝 Get support from your peers.  

There are thousands of students all over the world who are preparing for their AP exams just like you! Join  Rooms  🤝 to chat, ask questions, and meet other students who are also studying for the spring exams. You can even build study groups and review material together! 

2024 AP Lang Study Guide

🚧 unit 1 foundations of rhetoric: analysis of the rhetorical situation and claims ., big takeaways:.

Unit 1 is an introductory unit that lays the foundations for the reading skills associated with how to understand and analyze complex texts. Skills here include identifying the ASPECTS of a text, analyzing the claim given and the evidence used to support that claim, and determining the function of the “chunks” in the argument. Because the content in this unit is very foundational, it is looped throughout the rest of the course instruction.

Definitely do this:

📚 Read these study guides:

Unit 1 Overview: Claims , Reasoning , and Evidence

1.1 Identifying the purpose and intended audience of a text

1.2 Examining how evidence supports a claim

1.3 Developing paragraphs as part of an effective argument

🎥 Watch these videos:

College Board’s Instructional Video: Overview of The Rhetorical Situation .

Fiveable’s How to Read Like an AP Student .

Rhetorical Analysis Thesis Statements  

Rhetorical Analysis Body Paragraphs

✍️ Practice:

Use the Fiveable ASPECTS Guidesheet to help you break down a complex text.

🗺 Can you identify these rhetorical devices?

You won’t be asked to name drop on the exam, but it can be helpful to use devices when discussing strategies. Try this Quizlet to help prepare.

Unit 2 Foundations of Argument: Analysis of an author’s choices in appeals and evidence

Unit 2 is an introductory unit that builds onto the foundations of rhetorical ASPECTS and moves toward planning and writing your own arguments. This unit focuses on the relationships between subject, speaker, and message, including examination of the structure and purpose of the given argument. The unit then moves into the developing thesis statements and building your own arguments with a clear line of reasoning .

Unit 2 Overview: Organizing Information for a Specific Audience

2.1 Analyzing audience and its relationship to the purpose of an argument

2.2 Building an argument with relevant and strategic evidence

2.3 Developing thesis statements

2.4 Developing structure and integrating evidence to reflect a line of reasoning

College Board’s Instructional Video: Identify Rhetorical Situation in a Pre 20th Century Text .

Fiveable’s video on How to Find Rhetorical Devices  

📰 Check out these articles:

Here’s a list of recommended rhetorical devices with definitions and examples!

Use the Fiveable Rhetorical Precis Guidesheet to help you break down a complex text.

🗺 Can you identify these elements of practical argument?

You won’t be asked to name drop of the exam, but it can be helpful to use devices when discussing strategies. Try this Quizlet to help prepare.

👥 Unit 3 Confluence: Synthesis of multiple sources in argumentation

Unit 3 approaches multiple perspectives in argument through the lens of synthesis (that’s FRQ 1). In this study, you learn to identify effective and faulty reasoning while integrating a variety of evidence from credible resources that is properly cited in an original text.

Unit 3 Overview: Perspectives and How Arguments Relate

3.1 Interpreting character description and perspective

3.2 Identifying and avoiding flawed lines of reasoning

3.3 Introducing and integrating sources and evidence

3.4 Using sufficient evidence for an argument

3.5 Attributing and citing references

3.6 Developing parts of a text with cause-effect and narrative methods

Fiveable’s Introduction into Synthesis Essays and How to Begin Your Argument

College Board’s Instructional Video: Complexity in Argument .

🗺 Can you identify these elements of synthesis?

👀 Unit 4 Reasoning : Analysis of argument from introduction to conclusion

Unit 4 includes a greater depth of focus on the writing of effective arguments -- the line of reasoning created in the introduction, built with modes of discourse, and strengthened in the conclusion. An important note about these skills of argumentation is that they build toward all parts of every FRQ. 

Unit 4 Overview: How writers develop arguments, intros, and conclusion

4.1 Developing and connecting thesis statements and lines of reasoning

4.2 Developing introductions and conclusions

4.3 Adjusting an argument to address new evidence

College Board’s Instructional Video: Understanding a Line of Reasoning .

Fiveable’s Effective Annotations .

Try Fiveable’s Guide to LOR Body Paragraphs .

🗺 Can you identify the rhetorical modes?

You won’t be asked to name drop them on the exam, but it can be helpful to use devices when discussing strategies. Try this Quizlet to help prepare.

🧐 Unit 5 Commentary and Analysis: Analysis of complex argument and intentional rhetoric

In Unit 5, the skills look at the minutiae involved in argumentation: development of the line of reasoning that produces strong commentary and maintains the primary claim through all parts of the writing. To achieve these goals, this unit includes a focus on transitions , modifiers , and qualifications for argumentative perspective .  

Unit 5 Overview

5.1 Maintaining ideas throughout an argument

5.2 Developing commentary throughout paragraphs

5.3 Using modifiers to qualify an argument and convey perspective

5.4 Using transitions

Fiveable’s video on How to Improve Analysis Part 1 and Part 2

As well as how to Embed Quotes into Body Paragraphs  

Rhetorical Analysis Body Paragraphs  

Synthesis Essay Body Paragraphs  

Argument Essay Body Paragraphs

Tara Seale’s adaptation for Creating a Line of Reasoning .

🏃‍♂️ Unit 6 Rhetorical Risks: Analysis of multiple perspectives , bias , and shifts with new evidence

In Unit 6, you will notice a direct link building on the ideas of Unit 3 as this instruction looks at position and perspectives while synthesizing information strategically to support a claim.  For greater depth, this unit moves to modify a current argument to include new evidence .

Unit 6 Overview: Position, Perspective , and Bias

6.1 Incorporating multiple perspectives strategically into an argument

6.2 Recognizing and accounting for bias

6.3 Adjusting an argument to new evidence

6.4 Analyzing tone and shifts in tone

College Board’s Instructional Video: Creating a Nuanced Argument .

Fiveable’s video on Tracking an Author’s Argument  

🚀 Unit 7 Complex Argumentation: Analysis of effective arguments, including concession and refutation

The skills of Unit 7 are about putting all units of study together to look at the complexity of a given argument and the effectiveness of the pieces built into that argument.  Though many teachers will have addressed counterarguments, concessions, and refutations before reaching this unit, those skills are highly scrutinized in this segment of learning.

Unit 7 Overview: Successful and Unsuccessful Arguments

7.1 Examining complexities in issues

7.2 Considering how words, phrases, and clauses can modify and limit an argument

7.3 Examining how counterargument or alternative perspectives affect an argument

7.4 Exploring how sentence development affects an argument

Fiveable’s video on Arguments and Counterarguments  

College Board’s Instructional Video: How Argument Demonstrates Understanding .

Check your progress with Fiveable’s AP Language Skills Matrix .

📝 Unit 8 Style: Analysis of how style influences the audience movement

Unit 8 covers how to understand the influence style has on the audience , and the purpose behind each decision. By analyzing these various tactics, students are able to understand the author’s audience , and how to effectively persuade them. Style is an important part in connecting the rest of the course and understanding how the rhetorical choices and devices are used to accomplish a purpose .

Unit 8 Overview: Stylistic Choices

8.1 Choosing comparisons based on an audience

8.2 Considering how sentence development and word choice affect how the writer is perceived by an audience

8.3 Considering how all choices made in an argument affect the audience

8.4 Considering how style affects an argument

Fiveable’s Analysis of the Mindset of the Audience

College Board’s Instructional video: Analyzing and Understanding the Audience

College Board’s explanation of Elements and Context for Style  

Review this quizlet on Elements of Style for more practice.

✏️ Unit 9 Craft: Creation of your own complex argument with synthesis and rhetoric

The final unit of AP Language and Composition covers how to effectively form your own arguments by acknowledging and understanding complexities to create a nuanced and sophisticated argument. It focuses on your ability to comprehend and connect multiple sources to create a well reasoned, and detailed argument as well as how to add in your own rhetorical devices and choices to make your writing more persuasive and effective.

Unit 9 Overview: Developing a Complex Argument

9.1 Strategically conceding, rebutting, or refuting information

9.2 Crafting an argument through stylistic choices like word choice and description

Fiveable’s video on Creating your own Synthesis Arguments

College Board’s video on Complexities within Arguments and How to Create a Nuanced Argument

Key Terms to Review ( 38 )

Argument Structure

Author's Message

Cause-Effect Method

Comparisons

Conclusions

Counterargument

Introductions

Line of Reasoning

Multiple Perspectives

Narrative Method

Objective Reasoning

Perspective

Qualifications

Rhetorical Choices

Rhetorical Situation

Sentence Development

Stylistic Choices

Subjective Reasoning

Textual Evidence

Thesis Development

Thesis Statement

Tone Shifts

Transitions

Word Choice

Fiveable

Stay Connected

© 2024 Fiveable Inc. All rights reserved.

AP® and SAT® are trademarks registered by the College Board, which is not affiliated with, and does not endorse this website.

logo-type-white

AP® English Language

How to master ap® english language arguing.

  • The Albert Team
  • Last Updated On: March 1, 2022

how_to_master_AP® English language arguing

Acing the AP® English Language and Composition exam is no easy feat, but it can definitely be accomplished. Lang represents most high school students’ first foray into the world of AP® English exams, as it is traditionally taught before AP® English Literature and Composition. Between the two AP® English exams, it is definitely the easier test, as it is more skills-based than the Lit test and therefore more straightforward to master.

Don’t be fooled by the fact that this is an English class, or by the fact that your teacher may (and likely will) decide to pile novel after novel onto your syllabus. The trick to doing well on this exam does not hinge upon your ability to manipulate literary devices such as similies and metaphors. Memorizing the plots of novels and scouring cram books will be of limited use. In the end, doing well on AP® Lang will actually mean becoming a master of argumentation, plain and simple. Instead, reading, understanding, and practicing the usage of argumentative tactics will get you a 5.

There are of course two parts to this exam – essays and multiple-choice. 45% of the exam is actually purely multiple-choice based. This is the part you will need the cram book for the most. Practice test after practice test is the key, as the exam tests a large range of technical terminology related to argumentation. For both the multiple choice and the essay questions, it will be important to understand such terms and have them memorized. If your teacher hasn’t made up a list of such terms for you to study, there are plenty on the Internet and they also feature in cram books.

While cram books are important for taking practice multiple-choice exams (as on other exams, there are only so many variations you can ask on similar questions year to year, so taking as many exams as possible and reviewing the answers is highly recommended), they are of limited importance for this exam’s three essay questions. I hardly looked at my cram book except insofar as I wanted to review some key terms and go over exam questions. Don’t panic, though, there is definitely a plan for doing well on the essays, even if it is not as straightforward as plunking down with a cram book for a month. I would recommend reading great arguments, identifying their theses, the rhetorical strengths and strategies used within them (such as paradoxes, ethos, pathos) as well as weaknesses (such as straw man arguments and ad hominem fallacies). My experience dissecting famous works of argumentation such as Jonathan Swift’s A Modest Proposal and Mary Wollstencraft’s Vindication of the Rights of Women was invaluable on exam day.

Once you’ve prepared sufficiently for the exam by reading the writings of great masters of argumentation, the task of actually writing an essay is at hand. While there is a time and a place for deeply creative writing, acing this section of the AP® exam really comes down to how well you’ve mastered a formula, which I will outline below:

Introduction paragraph

This paragraph is key because it is your first impression on your essay reader. It is crucial to open strong.

  • Hook and/or Background

o   Include some background/historical information to open the first paragraph, along with or instead of an interesting “hook” to get your reader’s attention

  • Argument/thesis

o   This is perhaps the most crucial part of this essay. While the thesis is always important, it is especially crucial to make a definitive statement that you are capable of defending with the evidence.

o   Key tips to remember :

  • There are no “wrong” theses, only poorly defended ones. Make sure that whatever you say, you root it firmly in the evidence.
  • Pick a side. You don’t have to actually agree with it as long as it is, once again, firmly rooted in the evidence .

o   This is where you ease the reader into the structure of your paper. While you want to make sure you discuss argumentative devices such as ad-hominem attacks and paradoxes, you want to be careful not to turn your essay into a list of terms. This makes for a choppy, amateurish paper that will not merit full points.

o   Key tip : pick a flowing, appealing structure for your paper. You can build your argument (and discuss rhetorical terms) by chronologically dissecting an issue or narrative.

  • Note that you do not need to list all the strategies. You just need to make it clear how the paper will flow and how you will support your thesis.

o   Key tip : the very best essays make concessions to the other side of the argument. You can get the concession over with at the beginning or the end of the paper, just make sure that when you make a reasonable concession it does not overshadow your main argument.

First body paragraph + argument

AP® English Language paragraph

Each body paragraph should be at least five-eight sentences long and should include 1-2 pieces of evidence to support your argument.

  • First topic sentence/transition from thesis

o   Make sure you effectively transition from the thesis to the first piece of evidence. It can be a simple transitional sentence, but it is always important to remember to transition.

  • Set-up first piece of argument

o   Remember, you are trying to create an eloquent, easy to understand experience. Briefly situate the reader in the context of the argument you are discussing.

  • First piece of evidence

o   This can be a paraphrased excerpt or a direct quote.

  • Analyze first piece of evidence

o   This is crucial: do not merely state evidence/terms, rather, analyze them and offer something new .

  • Transition to second piece of evidence or the next paragraph

o   This depends on how many pieces of evidence fit in with this type of argumentative issue/device. It’s important to resist the urge to “megagraph” aka, the creation of huge, meandering paragraphs chockfull of evidence. If you do this, your meaning will get lost. Instead, stick with six succinct paragraphs with selectively chosen, strong evidence that supports your thesis and shows what you know.

While this may seem like a lot to tackle, if you break your studying up into at least a month’s worth of time you will be golden. The key, just like any other AP® exam, is practice. Practice this formula, memorize those terms, take as many practice exams as you can, and you will be good to go!

By the way, you should check out Albert.io for your AP® English Language review. We have hundreds of AP® English Language practice questions written just for you!

Interested in a school license?​

2 thoughts on “how to master ap® english language arguing”.

Thanks for this helpful information.

You’re welcome, Cassandra! Good luck!

Comments are closed.

Popular Posts

AP® Physics I score calculator

AP® Score Calculators

Simulate how different MCQ and FRQ scores translate into AP® scores

argumentative essay prompts for ap lang

AP® Review Guides

The ultimate review guides for AP® subjects to help you plan and structure your prep.

argumentative essay prompts for ap lang

Core Subject Review Guides

Review the most important topics in Physics and Algebra 1 .

argumentative essay prompts for ap lang

SAT® Score Calculator

See how scores on each section impacts your overall SAT® score

argumentative essay prompts for ap lang

ACT® Score Calculator

See how scores on each section impacts your overall ACT® score

argumentative essay prompts for ap lang

Grammar Review Hub

Comprehensive review of grammar skills

argumentative essay prompts for ap lang

AP® Posters

Download updated posters summarizing the main topics and structure for each AP® exam.

Interested in a school license?

argumentative essay prompts for ap lang

Bring Albert to your school and empower all teachers with the world's best question bank for: ➜ SAT® & ACT® ➜ AP® ➜ ELA, Math, Science, & Social Studies aligned to state standards ➜ State assessments Options for teachers, schools, and districts.

PrepScholar

Choose Your Test

Sat / act prep online guides and tips, how to write a perfect synthesis essay for the ap language exam.

author image

Advanced Placement (AP)

body-pencil-sharpen-notebook-1

If you're planning to take the AP Language (or AP Lang) exam , you might already know that 55% of your overall exam score will be based on three essays. The first of the three essays you'll have to write on the AP Language exam is called the "synthesis essay." If you want to earn full points on this portion of the AP Lang Exam, you need to know what a synthesis essay is and what skills are assessed by the AP Lang synthesis essay.

In this article, we'll explain the different aspects of the AP Lang synthesis essay, including what skills you need to demonstrate in your synthesis essay response in order to achieve a good score. We'll also give you a full breakdown of a real AP Lang Synthesis Essay prompt, provide an analysis of an AP Lang synthesis essay example, and give you four tips for how to write a synthesis essay.

Let's get started by taking a closer look at how the AP Lang synthesis essay works!

Synthesis Essay AP Lang: What It Is and How It Works

The AP Lang synthesis essay is the first of three essays included in the Free Response section of the AP Lang exam.

The AP Lang synthesis essay portion of the Free Response section lasts for one hour total . This hour consists of a recommended 15 minute reading period and a 40 minute writing period. Keep in mind that these time allotments are merely recommendations, and that exam takers can parse out the allotted 60 minutes to complete the synthesis essay however they choose.

Now, here's what the structure of the AP Lang synthesis essay looks like. The exam presents six to seven sources that are organized around a specific topic (like alternative energy or eminent domain, which are both past synthesis exam topics).

Of these six to seven sources, at least two are visual , including at least one quantitative source (like a graph or pie chart, for example). The remaining four to five sources are print text-based, and each one contains approximately 500 words.

In addition to six to seven sources, the AP Lang exam provides a written prompt that consists of three paragraphs. The prompt will briefly explain the essay topic, then present a claim that students will respond to in an essay that synthesizes material from at least three of the sources provided.

Here's an example prompt provided by the College Board:

Directions : The following prompt is based on the accompanying six sources.

This question requires you to integrate a variety of sources into a coherent, well-written essay. Refer to the sources to support your position; avoid mere paraphrase or summary. Your argument should be central; the sources should support this argument .

Remember to attribute both direct and indirect citations.

Introduction

Television has been influential in United States presidential elections since the 1960's. But just what is this influence, and how has it affected who is elected? Has it made elections fairer and more accessible, or has it moved candidates from pursuing issues to pursuing image?

Read the following sources (including any introductory information) carefully. Then, in an essay that synthesizes at least three of the sources for support, take a position that defends, challenges, or qualifies the claim that television has had a positive impact on presidential elections.

Refer to the sources as Source A, Source B, etc.; titles are included for your convenience.

Source A (Campbell) Source B (Hart and Triece) Source C (Menand) Source D (Chart) Source E (Ranney) Source F (Koppel)

Like we mentioned earlier, this prompt gives you a topic — which it briefly explains — then asks you to take a position. In this case, you'll have to choose a stance on whether television has positively or negatively affected U.S. elections. You're also given six sources to evaluate and use in your response. Now that you have everything you need, now your job is to write an amazing synthesis essay.

But what does "synthesize" mean, exactly? According to the CollegeBoard, when an essay prompt asks you to synthesize, it means that you should "combine different perspectives from sources to form a support of a coherent position" in writing. In other words, a synthesis essay asks you to state your claim on a topic, then highlight the relationships between several sources that support your claim on that topic. Additionally, you'll need to cite specific evidence from your sources to prove your point.

The synthesis essay counts for six of the total points on the AP Lang exam . Students can receive 0-1 points for writing a thesis statement in the essay, 0-4 based on incorporation of evidence and commentary, and 0-1 points based on sophistication of thought and demonstrated complex understanding of the topic.

You'll be evaluated based on how effectively you do the following in your AP Lang synthesis essay:

Write a thesis that responds to the exam prompt with a defensible position

Provide specific evidence that to support all claims in your line of reasoning from at least three of the sources provided, and clearly and consistently explain how the evidence you include supports your line of reasoning

Demonstrate sophistication of thought by either crafting a thoughtful argument, situating the argument in a broader context, explaining the limitations of an argument

Make rhetorical choices that strengthen your argument and/or employ a vivid and persuasive style throughout your essay.

If your synthesis essay meets the criteria above, then there's a good chance you'll score well on this portion of the AP Lang exam!

If you're looking for even more information on scoring, the College Board has posted the AP Lang Free Response grading rubric on its website. ( You can find it here. ) We recommend taking a close look at it since it includes additional details about the synthesis essay scoring.

body-chisel-break-apart

Don't be intimidated...we're going to teach you how to break down even the hardest AP synthesis essay prompt.

Full Breakdown of a Real AP Lang Synthesis Essay Prompt

In this section, we'll teach you how to analyze and respond to a synthesis essay prompt in five easy steps, including suggested time frames for each step of the process.

Step 1: Analyze the Prompt

The very first thing to do when the clock starts running is read and analyze the prompt. To demonstrate how to do this, we'll look at the sample AP Lang synthesis essay prompt below. This prompt comes straight from the 2018 AP Lang exam:

Eminent domain is the power governments have to acquire property from private owners for public use. The rationale behind eminent domain is that governments have greater legal authority over lands within their dominion than do private owners. Eminent domain has been instituted in one way or another throughout the world for hundreds of years.

Carefully read the following six sources, including the introductory information for each source. Then synthesize material from at least three of the sources and incorporate it into a coherent, well-developed essay that defends, challenges, or qualifies the notion that eminent domain is productive and beneficial.

Your argument should be the focus of your essay. Use the sources to develop your argument and explain the reasoning for it. Avoid merely summarizing the sources. Indicate clearly which sources you are drawing from, whether through direct quotation, paraphrase, or summary. You may cite the sources as Source A, Source B, etc., or by using the descriptions in parentheses.

On first read, you might be nervous about how to answer this prompt...especially if you don't know what eminent domain is! But if you break the prompt down into chunks, you'll be able to figure out what the prompt is asking you to do in no time flat.

To get a full understanding of what this prompt wants you to do, you need to identify the most important details in this prompt, paragraph by paragraph. Here's what each paragraph is asking you to do:

  • Paragraph 1: The prompt presents and briefly explains the topic that you'll be writing your synthesis essay about. That topic is the concept of eminent domain.
  • Paragraph 2: The prompt presents a specific claim about the concept of eminent domain in this paragraph: Eminent domain is productive and beneficial. This paragraph instructs you to decide whether you want to defend, challenge, or qualify that claim in your synthesis essay , and use material from at least three of the sources provided in order to do so.
  • Paragraph 3: In the last paragraph of the prompt, the exam gives you clear instructions about how to approach writing your synthesis essay . First, make your argument the focus of the essay. Second, use material from at least three of the sources to develop and explain your argument. Third, provide commentary on the material you include, and provide proper citations when you incorporate quotations, paraphrases, or summaries from the sources provided.

So basically, you'll have to agree with, disagree with, or qualify the claim stated in the prompt, then use at least three sources substantiate your answer. Since you probably don't know much about eminent domain, you'll probably decide on your position after you read the provided sources.

To make good use of your time on the exam, you should spend around 2 minutes reading the prompt and making note of what it's asking you to do. That will leave you plenty of time to read the sources provided, which is the next step to writing a synthesis essay.

Step 2: Read the Sources Carefully

After you closely read the prompt and make note of the most important details, you need to read all of the sources provided. It's tempting to skip one or two sources to save time--but we recommend you don't do this. That's because you'll need a thorough understanding of the topic before you can accurately address the prompt!

For the sample exam prompt included above, there are six sources provided. We're not going to include all of the sources in this article, but you can view the six sources from this question on the 2018 AP Lang exam here . The sources include five print-text sources and one visual source, which is a cartoon.

As you read the sources, it's important to read quickly and carefully. Don't rush! Keep your pencil in hand to quickly mark important passages that you might want to use as evidence in your synthesis. While you're reading the sources and marking passages, you want to think about how the information you're reading influences your stance on the issue (in this case, eminent domain).

When you finish reading, take a few seconds to summarize, in a phrase or sentence, whether the source defends, challenges, or qualifies whether eminent domain is beneficial (which is the claim in the prompt) . Though it might not feel like you have time for this, it's important to give yourself these notes about each source so you know how you can use each one as evidence in your essay.

Here's what we mean: say you want to challenge the idea that eminent domain is useful. If you've jotted down notes about each source and what it's saying, it will be easier for you to pull the relevant information into your outline and your essay.

So how much time should you spend reading the provided sources? The AP Lang exam recommends taking 15 minutes to read the sources . If you spend around two of those minutes reading and breaking down the essay prompt, it makes sense to spend the remaining 13 minutes reading and annotating the sources.

If you finish reading and annotating early, you can always move on to drafting your synthesis essay. But make sure you're taking your time and reading carefully! It's better to use a little extra time reading and understanding the sources now so that you don't have to go back and re-read the sources later.

body-weightlifting-lift-strong

A strong thesis will do a lot of heavy lifting in your essay. (See what we did there?)

Step 3: Write a Strong Thesis Statement

After you've analyzed the prompt and thoroughly read the sources, the next thing you need to do in order to write a good synthesis essay is write a strong thesis statement .

The great news about writing a thesis statement for this synthesis essay is that you have all the tools you need to do it at your fingertips. All you have to do in order to write your thesis statement is decide what your stance is in relationship to the topic provided.

In the example prompt provided earlier, you're essentially given three choices for how to frame your thesis statement: you can either defend, challenge, or qualify a claim that's been provided by the prompt, that eminent domain is productive and beneficial . Here's what that means for each option:

If you choose to defend the claim, your job will be to prove that the claim is correct . In this case, you'll have to show that eminent domain is a good thing.

If you choose to challenge the claim, you'll argue that the claim is incorrect. In other words, you'll argue that eminent domain isn't productive or beneficial.

If you choose to qualify, that means you'll agree with part of the claim, but disagree with another part of the claim. For instance, you may argue that eminent domain can be a productive tool for governments, but it's not beneficial for property owners. Or maybe you argue that eminent domain is useful in certain circumstances, but not in others.

When you decide whether you want your synthesis essay to defend, challenge, or qualify that claim, you need to convey that stance clearly in your thesis statement. You want to avoid simply restating the claim provided in the prompt, summarizing the issue without making a coherent claim, or writing a thesis that doesn't respond to the prompt.

Here's an example of a thesis statement that received full points on the eminent domain synthesis essay:

Although eminent domain can be misused to benefit private interests at the expense of citizens, it is a vital tool of any government that intends to have any influence on the land it governs beyond that of written law.

This thesis statement received full points because it states a defensible position and establishes a line of reasoning on the issue of eminent domain. It states the author's position (that some parts of eminent domain are good, but others are bad), then goes on to explain why the author thinks that (it's good because it allows the government to do its job, but it's bad because the government can misuse its power.)

Because this example thesis statement states a defensible position and establishes a line of reasoning, it can be elaborated upon in the body of the essay through sub-claims, supporting evidence, and commentary. And a solid argument is key to getting a six on your synthesis essay for AP Lang!

Looking for help studying for your AP exam?

Our one-on-one online AP tutoring services can help you prepare for your AP exams. Get matched with a top tutor who got a high score on the exam you're studying for!

Get a 5 On Your AP Exam

Step 4: Create a Bare-Bones Essay Outline

Once you've got your thesis statement drafted, you have the foundation you need to develop a bare bones outline for your synthesis essay. Developing an outline might seem like it's a waste of your precious time, but if you develop your outline well, it will actually save you time when you start writing your essay.

With that in mind, we recommend spending 5 to 10 minutes outlining your synthesis essay . If you use a bare-bones outline like the one below, labeling each piece of content that you need to include in your essay draft, you should be able to develop out the most important pieces of the synthesis before you even draft the actual essay.

To help you see how this can work on test day, we've created a sample outline for you. You can even memorize this outline to help you out on test day! In the outline below, you'll find places to fill in a thesis statement, body paragraph topic sentences, evidence from the sources provided, and commentary :

  • Present the context surrounding the essay topic in a couple of sentences (this is a good place to use what you learned about the major opinions or controversies about the topic from reading your sources).
  • Write a straightforward, clear, and concise thesis statement that presents your stance on the topic
  • Topic sentence presenting first supporting point or claim
  • Evidence #1
  • Commentary on Evidence #1
  • Evidence #2 (if needed)
  • Commentary on Evidence #2 (if needed)
  • Topic sentence presenting second supporting point or claim
  • Topic sentence presenting three supporting point or claim
  • Sums up the main line of reasoning that you developed and defended throughout the essay
  • Reiterates the thesis statement

Taking the time to develop these crucial pieces of the synthesis in a bare-bones outline will give you a map for your final essay. Once you have a map, writing the essay will be much easier.

Step 5: Draft Your Essay Response

The great thing about taking a few minutes to develop an outline is that you can develop it out into your essay draft. After you take about 5 to 10 minutes to outline your synthesis essay, you can use the remaining 30 to 35 minutes to draft your essay and review it.

Since you'll outline your essay before you start drafting, writing the essay should be pretty straightforward. You'll already know how many paragraphs you're going to write, what the topic of each paragraph will be, and what quotations, paraphrases, or summaries you're going to include in each paragraph from the sources provided. You'll just have to fill in one of the most important parts of your synthesis—your commentary.

Commentaries are your explanation of why your evidence supports the argument you've outlined in your thesis. Your commentary is where you actually make your argument, which is why it's such a critical part of your synthesis essay.

When thinking about what to say in your commentary, remember one thing the AP Lang synthesis essay prompt specifies: don't just summarize the sources. Instead, as you provide commentary on the evidence you incorporate, you need to explain how that evidence supports or undermines your thesis statement . You should include commentary that offers a thoughtful or novel perspective on the evidence from your sources to develop your argument.

One very important thing to remember as you draft out your essay is to cite your sources. The AP Lang exam synthesis essay prompt indicates that you can use generic labels for the sources provided (e.g. "Source 1," "Source 2," "Source 3," etc.). The exam prompt will indicate which label corresponds with which source, so you'll need to make sure you pay attention and cite sources accurately. You can cite your sources in the sentence where you introduce a quote, summary, or paraphrase, or you can use a parenthetical citation. Citing your sources affects your score on the synthesis essay, so remembering to do this is important.

body-green-arrow-down

Keep reading for a real-life example of a great AP synthesis essay response!

Real-Life AP Synthesis Essay Example and Analysis

If you're still wondering how to write a synthesis essay, examples of real essays from past AP Lang exams can make things clearer. These real-life student AP synthesis essay responses can be great for helping you understand how to write a synthesis essay that will knock the graders' socks off .

While there are multiple essay examples online, we've chosen one to take a closer look at. We're going to give you a brief analysis of one of these example student synthesis essays from the 2019 AP Lang Exam below!

Example Synthesis Essay AP Lang Response

To get started, let's look at the official prompt for the 2019 synthesis essay:

In response to our society's increasing demand for energy, large-scale wind power has drawn attention from governments and consumers as a potential alternative to traditional materials that fuel our power grids, such as coal, oil, natural gas, water, or even newer sources such as nuclear or solar power. Yet the establishment of large-scale, commercial-grade wind farms is often the subject of controversy for a variety of reasons.

Carefully read the six sources, found on the AP English Language and Composition 2019 Exam (Question 1), including the introductory information for each source. Write an essay that synthesizes material from at least three of the sources and develops your position on the most important factors that an individual or agency should consider when deciding whether to establish a wind farm.

Source A (photo) Source B (Layton) Source C (Seltenrich) Source D (Brown) Source E (Rule) Source F (Molla)

In your response you should do the following:

  • Respond to the prompt with a thesis presents a defensible position.
  • Select and use evidence from at least 3 of the provided sources to support your line of reasoning. Indicate clearly the sources used through direct quotation, paraphrase, or summary. Sources may be cited as Source A, Source B, etc., or by using the description in parentheses.
  • Explain how the evidence supports your line of reasoning.
  • Use appropriate grammar and punctuation in communicating your argument.

Now that you know exactly what the prompt asked students to do on the 2019 AP Lang synthesis essay, here's an AP Lang synthesis essay example, written by a real student on the AP Lang exam in 2019:

[1] The situation has been known for years, and still very little is being done: alternative power is the only way to reliably power the changing world. The draw of power coming from industry and private life is overwhelming current sources of non-renewable power, and with dwindling supplies of fossil fuels, it is merely a matter of time before coal and gas fuel plants are no longer in operation. So one viable alternative is wind power. But as with all things, there are pros and cons. The main factors for power companies to consider when building wind farms are environmental boon, aesthetic, and economic factors.

[2] The environmental benefits of using wind power are well-known and proven. Wind power is, as qualified by Source B, undeniably clean and renewable. From their production requiring very little in the way of dangerous materials to their lack of fuel, besides that which occurs naturally, wind power is by far one of the least environmentally impactful sources of power available. In addition, wind power by way of gearbox and advanced blade materials, has the highest percentage of energy retention. According to Source F, wind power retains 1,164% of the energy put into the system – meaning that it increases the energy converted from fuel (wind) to electricity 10 times! No other method of electricity production is even half that efficient. The efficiency and clean nature of wind power are important to consider, especially because they contribute back to power companies economically.

[3] Economically, wind power is both a boon and a bone to electric companies and other users. For consumers, wind power is very cheap, leading to lower bills than from any other source. Consumers also get an indirect reimbursement by way of taxes (Source D). In one Texan town, McCamey, tax revenue increased 30% from a wind farm being erected in the town. This helps to finance improvements to the town. But, there is no doubt that wind power is also hurting the power companies. Although, as renewable power goes, wind is incredibly cheap, it is still significantly more expensive than fossil fuels. So, while it is helping to cut down on emissions, it costs electric companies more than traditional fossil fuel plants. While the general economic trend is positive, there are some setbacks which must be overcome before wind power can take over as truly more effective than fossil fuels.

[4] Aesthetics may be the greatest setback for power companies. Although there may be significant economic and environmental benefit to wind power, people will always fight to preserve pure, unspoiled land. Unfortunately, not much can be done to improve the visual aesthetics of the turbines. White paint is the most common choice because it "[is] associated with cleanliness." (Source E). But, this can make it stand out like a sore thumb, and make the gargantuan machines seem more out of place. The site can also not be altered because it affects generating capacity. Sound is almost worse of a concern because it interrupts personal productivity by interrupting people's sleep patterns. One thing for power companies to consider is working with turbine manufacturing to make the machines less aesthetically impactful, so as to garner greater public support.

[5] As with most things, wind power has no easy answer. It is the responsibility of the companies building them to weigh the benefits and the consequences. But, by balancing economics, efficiency, and aesthetics, power companies can create a solution which balances human impact with environmental preservation.

And that's an entire AP Lang synthesis essay example, written in response to a real AP Lang exam prompt! It's important to remember AP Lang exam synthesis essay prompts are always similarly structured and worded, and students often respond in around the same number of paragraphs as what you see in the example essay response above.

Next, let's analyze this example essay and talk about what it does effectively, where it could be improved upon, and what score past exam scorers awarded it.

To get started on an analysis of the sample synthesis essay, let's look at the scoring commentary provided by the College Board:

  • For development of thesis, the essay received 1 out of 1 possible points
  • For evidence and commentary, the essay received 4 out of 4 possible points
  • For sophistication of thought, the essay received 0 out of 1 possible points.

This means that the final score for this example essay was a 5 out of 6 possible points . Let's look more closely at the content of the example essay to figure out why it received this score breakdown.

Thesis Development

The thesis statement is one of the three main categories that is taken into consideration when you're awarded points on this portion of the exam. This sample essay received 1 out of 1 total points.

Now, here's why: the thesis statement clearly and concisely conveys a position on the topic presented in the prompt--alternative energy and wind power--and defines the most important factors that power companies should consider when deciding whether to establish a wind farm.

Evidence and Commentary

The second key category taken into consideration when synthesis exams are evaluated is incorporation of evidence and commentary. This sample received 4 out of 4 possible points for this portion of the synthesis essay. At bare minimum, this sample essay meets the requirement mentioned in the prompt that the writer incorporate evidence from at least three of the sources provided.

On top of that, the writer does a good job of connecting the incorporated evidence back to the claim made in the thesis statement through effective commentary. The commentary in this sample essay is effective because it goes beyond just summarizing what the provided sources say. Instead, it explains and analyzes the evidence presented in the selected sources and connects them back to supporting points the writer makes in each body paragraph.

Finally, the writer of the essay also received points for evidence and commentary because the writer developed and supported a consistent line of reasoning throughout the essay . This line of reasoning is summed up in the fourth paragraph in the following sentence: "One thing for power companies to consider is working with turbine manufacturing to make the machines less aesthetically impactful, so as to garner greater public support."

Because the writer did a good job consistently developing their argument and incorporating evidence, they received full marks in this category. So far, so good!

Sophistication of Thought

Now, we know that this essay received a score of 5 out of 6 total points, and the place where the writer lost a point was on the basis of sophistication of thought, for which the writer received 0 out of 1 points. That's because this sample essay makes several generalizations and vague claims where it could have instead made specific claims that support a more balanced argument.

For example, in the following sentence from the 5th paragraph of the sample essay, the writer misses the opportunity to state specific possibilities that power companies should consider for wind energy . Instead, the writer is ambiguous and non-committal, saying, "As with most things, wind power has no easy answer. It is the responsibility of the companies building them to weigh the benefits and consequences."

If the writer of this essay was interested in trying to get that 6th point on the synthesis essay response, they could consider making more specific claims. For instance, they could state the specific benefits and consequences power companies should consider when deciding whether to establish a wind farm. These could include things like environmental impacts, economic impacts, or even population density!

Despite losing one point in the last category, this example synthesis essay is a strong one. It's well-developed, thoughtfully written, and advances an argument on the exam topic using evidence and support throughout.

body-number-four-post-it-note

4 Tips for How to Write a Synthesis Essay

AP Lang is a timed exam, so you have to pick and choose what you want to focus on in the limited time you're given to write the synthesis essay. Keep reading to get our expert advice on what you should focus on during your exam.

Tip 1: Read the Prompt First

It may sound obvious, but when you're pressed for time, it's easy to get flustered. Just remember: when it comes time to write the synthesis essay, read the prompt first !

Why is it so important to read the prompt before you read the sources? Because when you're aware of what kind of question you're trying to answer, you'll be able to read the sources more strategically. The prompt will help give you a sense of what claims, points, facts, or opinions to be looking for as you read the sources.

Reading the sources without having read the prompt first is kind of like trying to drive while wearing a blindfold: you can probably do it, but it's likely not going to end well!

Tip 2: Make Notes While You Read

During the 15-minute reading period at the beginning of the synthesis essay, you'll be reading through the sources as quickly as you can. After all, you're probably anxious to start writing!

While it's definitely important to make good use of your time, it's also important to read closely enough that you understand your sources. Careful reading will allow you to identify parts of the sources that will help you support your thesis statement in your essay, too.

As you read the sources, consider marking helpful passages with a star or check mark in the margins of the exam so you know which parts of the text to quickly re-read as you form your synthesis essay. You might also consider summing up the key points or position of each source in a sentence or a few words when you finish reading each source during the reading period. Doing so will help you know where each source stands on the topic given and help you pick the three (or more!) that will bolster your synthesis argument.

Tip 3: Start With the Thesis Statement

If you don't start your synthesis essay with a strong thesis statement, it's going to be tough to write an effective synthesis essay. As soon as you finish reading and annotating the provided sources, the thing you want to do next is write a strong thesis statement.

According to the CollegeBoard grading guidelines for the AP Lang synthesis essay, a strong thesis statement will respond to the prompt— not restate or rephrase the prompt. A good thesis will take a clear, defensible position on the topic presented in the prompt and the sources.

In other words, to write a solid thesis statement to guide the rest of your synthesis essay, you need to think about your position on the topic at hand and then make a claim about the topic based on your position. This position will either be defending, challenging, or qualifying the claim made in the essay's prompt.

The defensible position that you establish in your thesis statement will guide your argument in the rest of the essay, so it's important to do this first. Once you have a strong thesis statement, you can begin outlining your essay.

Tip 4: Focus on Your Commentary

Writing thoughtful, original commentary that explains your argument and your sources is important. In fact, doing this well will earn you four points (out of a total of six)!

AP Lang provides six to seven sources for you on the exam, and you'll be expected to incorporate quotations, paraphrases, or summaries from at least three of those sources into your synthesis essay and interpret that evidence for the reader.

While incorporating evidence is very important, in order to get the extra point for "sophistication of thought" on the synthesis essay, it's important to spend more time thinking about your commentary on the evidence you choose to incorporate. The commentary is your chance to show original thinking, strong rhetorical skills, and clearly explain how the evidence you've included supports the stance you laid out in your thesis statement.

To earn the 6th possible point on the synthesis essay, make sure your commentary demonstrates a nuanced understanding of the source material, explains this nuanced understanding, and places the evidence incorporated from the sources in conversation with each other. To do this, make sure you're avoiding vague language. Be specific when you can, and always tie your commentary back to your thesis!

body-person-arrows-next

What's Next?

There's a lot more to the AP Language exam than just the synthesis essay. Be sure to check out our expert guide to the entire exam , then learn more about the tricky multiple choice section .

Is the AP Lang exam hard...or is it easy? See how it stacks up to other AP tests on our list of the hardest AP exams .

Did you know there are technically two English AP exams? You can learn more about the second English AP test, the AP Literature exam, in this article . And if you're confused about whether you should take the AP Lang or AP Lit test , we can help you make that decision, too.

Want to improve your SAT score by 160 points or your ACT score by 4 points? We've written a guide for each test about the top 5 strategies you must be using to have a shot at improving your score. Download it for free now:

Get eBook: 5 Tips for 160+ Points

Ashley Sufflé Robinson has a Ph.D. in 19th Century English Literature. As a content writer for PrepScholar, Ashley is passionate about giving college-bound students the in-depth information they need to get into the school of their dreams.

Student and Parent Forum

Our new student and parent forum, at ExpertHub.PrepScholar.com , allow you to interact with your peers and the PrepScholar staff. See how other students and parents are navigating high school, college, and the college admissions process. Ask questions; get answers.

Join the Conversation

Ask a Question Below

Have any questions about this article or other topics? Ask below and we'll reply!

Improve With Our Famous Guides

  • For All Students

The 5 Strategies You Must Be Using to Improve 160+ SAT Points

How to Get a Perfect 1600, by a Perfect Scorer

Series: How to Get 800 on Each SAT Section:

Score 800 on SAT Math

Score 800 on SAT Reading

Score 800 on SAT Writing

Series: How to Get to 600 on Each SAT Section:

Score 600 on SAT Math

Score 600 on SAT Reading

Score 600 on SAT Writing

Free Complete Official SAT Practice Tests

What SAT Target Score Should You Be Aiming For?

15 Strategies to Improve Your SAT Essay

The 5 Strategies You Must Be Using to Improve 4+ ACT Points

How to Get a Perfect 36 ACT, by a Perfect Scorer

Series: How to Get 36 on Each ACT Section:

36 on ACT English

36 on ACT Math

36 on ACT Reading

36 on ACT Science

Series: How to Get to 24 on Each ACT Section:

24 on ACT English

24 on ACT Math

24 on ACT Reading

24 on ACT Science

What ACT target score should you be aiming for?

ACT Vocabulary You Must Know

ACT Writing: 15 Tips to Raise Your Essay Score

How to Get Into Harvard and the Ivy League

How to Get a Perfect 4.0 GPA

How to Write an Amazing College Essay

What Exactly Are Colleges Looking For?

Is the ACT easier than the SAT? A Comprehensive Guide

Should you retake your SAT or ACT?

When should you take the SAT or ACT?

Stay Informed

argumentative essay prompts for ap lang

Get the latest articles and test prep tips!

Looking for Graduate School Test Prep?

Check out our top-rated graduate blogs here:

GRE Online Prep Blog

GMAT Online Prep Blog

TOEFL Online Prep Blog

Holly R. "I am absolutely overjoyed and cannot thank you enough for helping me!”

IMAGES

  1. How to Get a 6 on Argument FRQ in AP® English Language

    argumentative essay prompts for ap lang

  2. Incredible Ap Lang Argument Essay Prompts ~ Thatsnotus

    argumentative essay prompts for ap lang

  3. 010 Essay Example Ap Lang Argument Prompts Types Of Exam Study ~ Thatsnotus

    argumentative essay prompts for ap lang

  4. 😊 Ap lang argument essay example. 3 Ways to Score a 9 on an AP English

    argumentative essay prompts for ap lang

  5. Sample Argumentative Essay.doc

    argumentative essay prompts for ap lang

  6. AP Language and Composition: Argument Prompt

    argumentative essay prompts for ap lang

VIDEO

  1. Argumentative Writing

  2. The BEST Way to Break Down the Argument Prompt!

  3. AP LANG

  4. How to Integrate Quotes the BEST WAY When Writing About Poetry!

  5. Argument Essay Review

  6. The BEST Tool to Generate Evidence for Arguments!

COMMENTS

  1. AP English Language and Composition Past Exam Questions

    Download free-response questions from past exams along with scoring guidelines, sample responses from exam takers, and scoring distributions. If you are using assistive technology and need help accessing these PDFs in another format, contact Services for Students with Disabilities at 212-713-8333 or by email at [email protected]. Note ...

  2. How to Write the AP Lang Argument Essay (With Example)

    The AP Lang Argument Essay is one of three essays included in the written portion of the AP English Exam. The full AP English Exam is 3 hours and 15 minutes long, with the first 60 minutes dedicated to multiple-choice questions.

  3. How to Write the AP Lang Argument Essay + Examples

    2. Pick one side of the argument, but acknowledge the other side. When you write the essay, it's best if you pick one side of the debate and stick with it for the entire essay. All your evidence should be in support of that one side. However, in your introductory paragraph, as you introduce the debate, be sure to mention any merit the ...

  4. PDF AP English Language and Composition

    The argument prompt for this year's exam asked students to write an essay that argued their position on the value of striving for perfection. In their responses they were expected to respond to the prompt with a thesis that presented a defensible position, provide evidence to support their line of reasoning, explain how the evidence

  5. PDF AP Language Argument Prompts (some adaptions)

    Asher AP ELAC Past AP Language ARGUMENT Prompts (some adaptations) 2012 Consider the distinct perspective expressed in the following statements: "If you develop the absolute sense of certainty that powerful beliefs provide, then you can get yourself to accomplish virtually anything, including those things that other people are certain are impossible." (William Phelps )

  6. PDF AP English Language and Composition Question 3: Argument (2019) Sample

    AP English Language and Composition Question 3: Argument (2019) Sample Student Responses 5 [5] Despite being completely different people, a person in Texas should be worth the same as a person in Vermont. In the Declaration of Independence, Thomas Jefferson said that "all men were created equal," but the system he created is not.

  7. Crafting an Impressive Argumentative Essay for AP Lang

    Writing an argumentative essay for AP Language and Composition requires a strategic approach to effectively convey your perspective. Here's a guide to crafting an impressive argumentative essay: 1. Understand the Prompt: - Tip: Carefully read and analyze the prompt. Identify the key elements, including the task you are asked to perform and any ...

  8. PDF AP English Language and Composition

    In your response you should do the following: Respond to the prompt with a thesis that presents a defensible position. Provide evidence to support your line of reasoning. Explain how the evidence supports your line of reasoning. Use appropriate grammar and punctuation in communicating your argument. 2023 College Board.

  9. How to Craft an Argument for AP® English Language

    How the AP® English Language Persuasive Essay Works. Persuasion through essay writing is something you probably learned about a long time ago, but the AP® English Language Exam's persuasion essay requires some more specific tips. You will be given a prompt that may or may not reference a reading sample; it will ask you to then "defend ...

  10. Mastering the Argumentative Essay in AP Lang: Strategies and Examples

    By understanding the prompt, developing a clear thesis, providing relevant evidence, and utilizing persuasive techniques, students can create compelling argumentative essays in AP Lang. With practice and dedication, their writing skills will improve, leading to success in the AP Lang exam. Strategies for Crafting a Strong Argumentative Essay

  11. AP Lang Argument Essay: Evidence

    Argument FRQ. : An Argument FRQ (Free Response Question) is an essay question on the AP English Language exam that requires students to construct and defend an argument using evidence and rhetorical strategies. Audience. : The audience refers to the intended recipients or listeners of a message.

  12. AP English Language and Composition Exam

    Section II: Free Response. 3 Questions | 2 hours 15 minutes (includes a 15-minute reading period | 55% of Exam Score. Students write essays that respond to 3 free-response prompts from the following categories: Synthesis Question: After reading 6-7 texts about a topic (including visual and quantitative sources), students will compose an ...

  13. How to Get a 6 on Argument FRQ in AP® English Language

    Pick an opinion and stick to it. Choose one side of the argument and one clear claim to support all the way through. Craft a thesis statement. Your thesis should be clear, concise, and introduce the content of your essay. Craft a chronological argument. Make an argument that builds on its prior points.

  14. Expert Guide to the AP Language and Composition Exam

    The AP English Language and Composition Multiple-Choice. The multiple-choice section tests you on two main areas. The first is how well you can read and understand nonfiction passages for their use of rhetorical devices and tools. The second is how well you can "think like a writer" and make revisions to texts in composition questions.

  15. Tackling the AP English Language and Composition essays: part 1

    CC | Tackling the AP English Language and Composition essays: part 1. More than any other test, the AP English Language and Composition Exam is dominated by essays. Three timed essays—the Synthesis Essay, Rhetoric Essay, and Argument Essay—will take up most of your time on the exam, and count for more than fifty percent of your score.

  16. AP Lang Exam Guide

    1 argument essay. Scoring Rubric for the 2024 AP Lang Essays. Synthesis Essay. Thesis. 1 point for a defensible thesis that responds to the prompt. Evidence and Commentary. Max of 4 points for providing evidence from at least 3 sources that support the line of reasoning AND commentary that explains and analyzes the evidence. Sophistication

  17. How to Master AP® English Language Arguing

    In the end, doing well on AP® Lang will actually mean becoming a master of argumentation, plain and simple. Instead, reading, understanding, and practicing the usage of argumentative tactics will get you a 5. There are of course two parts to this exam - essays and multiple-choice. 45% of the exam is actually purely multiple-choice based.

  18. PDF MsEffie's List of Advanced Placement® Language and Composition Prompts

    Evaluate his argument. (Excerpt included) Analyze the rhetorical techniques Ellen Goodman uses to convey her attitude toward Phil, the subject of her piece, "The Company Man." After reading his paragraph, defend, challenge, or qualify James Baldwin's ideas about the importance of language as a "key to identity" and social acceptance.

  19. PDF AP English Language and Composition

    AP® English Language and Composition 2022 Scoring Guidelines. Argument Essay 6 points . Colin Powell, a four-star general and former United States secretary of state, wrote in his 1995 autobiography: "[W]e do not have the luxury of collecting information indefinitely. At some point, before we can have every possible fact in hand, we have to ...

  20. Every AP English Language and Composition Practice Exam

    The AP Language and Composition exam has two sections: a multiple-choice section with 45 questions, and a free-response section with three essay questions—one synthesis prompt, one analysis prompt, and one argument prompt. But not all AP Lang practice tests are like the real exam, and they aren't all of equal quality.

  21. PDF AP English Language & Composition Exam Prompts (1981 to 2017)

    detail, manipulation of language, and tone. Write a persuasive essay that defends, challenges, or qualifies the assertion that "For in much wisdom is much grief, and increase of knowledge is increase of sorrow" (Ecclesiastes). 1992 Analyze Queen Elizabeth I's diction, imagery, and sentence structure to achieve

  22. SAT® and AP® English Language essay prompts

    Click below to view a list of official SAT® Essay prompts. Then, click the title of each prompt to view the page or download a PDF of the text. 💡Some prompts below include links to an external resource. To include this link as your essay prompt when creating a writing assignment in NoRedInk, be sure to copy and paste the website's full ...

  23. How to Write a Perfect Synthesis Essay for the AP Language Exam

    The AP Lang exam synthesis essay prompt indicates that you can use generic labels for the sources provided (e.g. "Source 1," "Source 2," "Source 3," etc.). ... The defensible position that you establish in your thesis statement will guide your argument in the rest of the essay, so it's important to do this first. Once you have a strong thesis ...